Jump to content

duythanbg's Content

There have been 76 items by duythanbg (Search limited from 07-06-2020)



Sort by                Order  

#583533 Phương pháp học phương trình hàm

Posted by duythanbg on 21-08-2015 - 09:41 in Tài liệu, chuyên đề, phương pháp về Phương trình hàm

E nên chăm chỉ học và giải toán.

Toán sơ cấp không có phương pháp cụ thể giải từng bài đâu.

 

Tài liệu : 

 

Attached Files




#583526 CMR $a^{2}+b^{2}-abc$ là số chính phương

Posted by duythanbg on 21-08-2015 - 09:31 in Các dạng toán khác

Bài này là bài khá kinh điển cho NL Cực hạn .

Mình nhớ không nhầm thì là của tạp chí CRUX. 

 

Tài liệu NL Cực Hạn của thầy T N Dũng :

 

 

 

Attached Files




#583202 Chứng minh: $(1-xy)(1-yz)(1-zx)\geq 0$

Posted by duythanbg on 19-08-2015 - 21:00 in Bất đẳng thức - Cực trị

Lời giải : 

Dễ chứng minh : $xyz\geq 1$ do đó $yz\geq 1$

BĐT sẽ có dạng tương đương là : 

$(1-xy)(1-xz)\geq 0\Leftrightarrow 1-xy-xz+x(x+y+z-2)\geq 0 \Leftrightarrow x^2-2x+1\geq 0$

 

Hiển nhiên đúng.

 

( Chú là thằng thủ khoa lớp 10 Toán phải không ? Anh là tiền bối đây  :icon10: )




#582948 PT Hàm -Tuyển tập các bài toán sưu tầm từ Mathlinks.ro

Posted by duythanbg on 18-08-2015 - 21:29 in Phương trình hàm

Bài 47 là đề VNTST 2007

Bài 46: 

 

Cho x = 0 ta được : $f(f(y))=2y+f(0)$

Do đó f là song ánh Cho tiếp y = 0 ta được ngay $f(0)=0$

Do đó $f(x+f(y))=f(x)+2y=f(x)+f(f(y))$ suy ra ngay f là hàm cộng tính ( Do f toàn ánh ) 

 

Mà f liên tục nên $f(x)=kx$

Thử lại được $k=\sqrt{2}$




#582938 Chứng minh $X,Y,Z$ thẳng hàng

Posted by duythanbg on 18-08-2015 - 21:13 in Hình học

Bài 1 : 

Xem ở đây : http://www.artofprob...c6h30921p191489




#582932 $f(xf(y)+f(x))=2f(x)+xy$

Posted by duythanbg on 18-08-2015 - 21:08 in Phương trình hàm

$f:\mathbb{R}\rightarrow \mathbb{R}$

$f(xf(y)+f(x))=2f(x)+xy$




#582930 $f(x+yf(x))+f(xf(y)-y)= f(x)-f(y)+2xy$

Posted by duythanbg on 18-08-2015 - 21:05 in Phương trình hàm

Đây là bài 6 trong quyển 106 phương trình hàm 

 

Attached Files




#582926 CMR đường tròn Euler tiếp xúc với đường tròn nội tiếp và các đường tròn bàng...

Posted by duythanbg on 18-08-2015 - 20:58 in Hình học

Xét tam giác ABC với $(I),(I_{a}),(E)$ lần lượt là các đường tròn nội tiếp, bàng tiếp góc A và đường tròn Euler.

Gọi tiếp điểm của $(I),(I_{a})$ với BC là A' và A''

Gọi M,N,P là trung điểm của BC,CA,AB.

AI cắt BC tại I' 

Kẻ I'J vuông góc với ME.

 

Bạn hãy chứng minh :

Phép nghịch đảo : $N\tfrac{MA'^2}{M}$ biến $(I)\rightarrow (I_{a})$  và $(E)\rightarrow I'J$

Sau đó chứng minh : I'J đối xứng với BC qua phân giác của góc A 

Nên theo tính chất phép nghịch đảo suy ra ngay ĐPCM 




#582377 $AC$ vuông BD.

Posted by duythanbg on 16-08-2015 - 16:52 in Hình học

Lời giải :

 Gọi H,M,K,N là trung điểm AB,BC,CD,AD

Khi đó HMKN là hình bình hành ( ... )                                                                  (1)

Ta có : $\overrightarrow{AD}+\overrightarrow{BC}=2\overrightarrow{HK}$

Và $\overrightarrow{AB}+\overrightarrow{DC}=2\overrightarrow{NM}$

Do đó HK = MN                                                                                                    (2)

 

Từ (1),(2) suy ra : HMKN là hình chữ nhật , dễ dàng suy ra AC vuông BD




#582376 Tìm Min của $A= ab + 2bc + 2ca$

Posted by duythanbg on 16-08-2015 - 16:45 in Bất đẳng thức và cực trị

:mellow: mình nghĩ là nó ko có Min đâu  :icon10:




#582359 Tìm Min của $A= ab + 2bc + 2ca$

Posted by duythanbg on 16-08-2015 - 16:00 in Bất đẳng thức và cực trị

Ta có :

     $A=ab+2c(a+b)\leq \frac{(a+b)^2}{4}+2(a+b)(1-a-b)=\frac{-7}{4}t^2+2t\leq \frac{4}{7}$




#582342 Tìm vị trí của M sao cho HK lớn nhất

Posted by duythanbg on 16-08-2015 - 15:47 in Hình học

Ta có :

 

HK = AM . sin BAC

 

Do đó $HK max\Leftrightarrow AMmax$ khi và chỉ khi M đối xứng với A qua O.




#582129 CMR: MA = 3DA

Posted by duythanbg on 15-08-2015 - 20:21 in Hình học

Dưới góc nhìn của anh thì là DỄ.  :icon6:  :D 

 

( Nếu em biết định lý Menelaus thì nó chỉ là " Muỗi")

 

Lời giải :

Ta có các t/c đơn giản : OD = OE ; OA = 2OH                        (1)

Kẻ DK // AH

Do đó OH là đường trung bình của DEK suy ra DK = 2OH    (2)

 

(1),(2) suy ra  $\frac{DK}{AH}=\frac{2OH}{OH+2OH}=\frac{2}{3}$

Mà $\frac{MD}{MA}=\frac{DK}{AH}$            

 

=> ....




#565041 Phương pháp : Tổng các hàm số ( S.O.F)

Posted by duythanbg on 11-06-2015 - 21:57 in Bất đẳng thức - Cực trị

PP trên cũng áp dụng được với BĐT đối xứng. Mình sẽ viết sau.




#564998 Phương pháp : Tổng các hàm số ( S.O.F)

Posted by duythanbg on 11-06-2015 - 20:09 in Bất đẳng thức - Cực trị

           Do hạn chế bài viết nên mình sẽ chỉ đề cập đến BĐT Hoán vị vì phương pháp này giải các BĐT Hoán vị rõ ràng hơn, dễ hiểu hơn.

 

Ta có 2 định lý đơn giản sau : 

 

Định lý 1 : Nếu a và b là 2 số thỏa mãn : $a+b\geq 0$ thì một trong 2 số a,b phải không âm.

Định lý 2 : Nếu ta có  $f(a,b,c)\geq 0$ là một BĐT Hoán vị đúng  thì cũng tương đương với $f(a,c,b)\geq 0$ là một BĐT Hoán vị đúng

 

Từ đó ta có Định lý S.O.F

 

Để Chứng minh BĐT Hoán vị  $f(a,b,c)\geq 0$ ta sẽ chứng minh : $f(a,b,c)+f(a,c,b)\geq 0$

 

Sau đây là phần ứng dụng khá thú vị của S.O.F, nó sẽ giúp bạn hiểu rõ hơn.

 

BT1: ( IMO 1984)

Cho a,b,c là 3 cạnh của một tam giác, CMR : $a^2b(a-b)+b^2c(b-c)+c^2a(c-a)\geq 0$

 

Lời giải : 

Điều kiện a,b,c là 3 cạnh của một tam giác là không cần thiết.

Đặt $f(a,b,c)=a^2b(a-b)+b^2c(b-c)+c^2a(c-a)$

Ta có : với x,y,z là các số dương

$f(x,y,z)+f(x,z,y)=\sum (x^3y-x^2y^2+xy^3-x^2y^2)=\sum (xy(x-y)^2)\geq 0$

( Hiển nhiên đúng )

 

Do đó theo Định lý 1 : Một trong 2 biểu thức $f(x,y,z),f(x,z,y)$ phải không âm.

Nếu $f(x,y,z)\geq 0$ ta cho : $x=a,y=b,z=c$ suy ra ĐPCM.

Nếu $f(x,z,y)\geq 0$ ta cho : $x=a,y=c,z=b$ suy ra ĐPCM.

 

Nếu bạn đã hiều rồi thì ta không cần đặt thêm biến mới x,y,z nữa , ta sẽ sử dụng trực tiếp Định lý S.O.F luôn. Ta đến với bài toán nổi tiếng và khó sau :

 

BT2 : ( Vasile Cirtoaje) 

Cho a,b,c là các số dương. CMR : 

$(a^2+b^2+c^2)^2\geq 3(a^3b+b^3c+c^3a)$

 

Lời giải :

Ta chỉ cần chứng minh : 

$2(a^2+b^2+c^2)^2\geq 3\sum (a^3b+ab^3) \Leftrightarrow \sum (a^4+b^4+4a^2b^2-3a^3b-3ab^3)\geq 0 \Leftrightarrow \sum ((a^4+b^4-2a^2b^2)-(3a^3b+3ab^3-6a^2b^2))\geq 0\Leftrightarrow \sum (a-b)^2(a^2+b^2-ab)\geq 0$

 

Hiển nhiên đúng !!!!!

 

BT3: Cho a,b,c là các số thực dương. CMR :

$\sum \frac{a^3}{a+b}\geq \frac{a^2+b^2+c^2}{2}$

 

Lời giải : 

Ta chỉ cần chứng minh : 

$\sum \frac{a^3+b^3}{a+b}\geq a^2+b^2+c^2\Leftrightarrow \sum (a^2-ab+b^2)\geq a^2+b^2+c^2\Leftrightarrow a^2+b^2+c^2\geq ab+bc+ca$

 

Hiển nhiên đúng !

 

Các bạn có thể thấy sau khi sử dụng S.O.F thì BĐT hoán vị 3 biến ban đầu sẽ chuyển về đối xứng 3 biến, điều này gợi ý cho ta sử dụng phương pháp S.O.S

 

BT4 : (India 2002)

Cho a,b,c là 3 số dương. CMR : 

 

$\sum \frac{a}{b}\geq \sum \frac{a+c}{b+c}$

 

Lời giải : 

 

Hoàn toàn tương tự ta chỉ cần chứng minh :

$\sum (\frac{a}{b}-\frac{a+c}{b+c}+\frac{b}{a}-\frac{b+c}{a+c})\geq 0\Leftrightarrow \sum (\frac{c(a-b)}{b(b+c)}-\frac{c(a-b)}{a(a+c)})\geq 0\Leftrightarrow \sum \frac{c(a-b)^2(a+b+c)}{ab(a+c)(b+c)}\geq 0$

 

Đúng !

 

BT5 : (Vasile Cirtoaje)

Cho a,b,c là các số thực dương. CMR : 

$3(\frac{a}{b}+\frac{b}{c}+\frac{c}{a}-1)\geq 2(\frac{b}{a}+\frac{c}{b}+\frac{a}{c})$

 

Lời giải : 

Tương tự như trên, sau khi sử dụng S.O.F rồi giảm các biểu thức ở 2 vế ta đưa BĐT về dạng hiển nhiên sau : 

 

$\sum \left ( \frac{a}{b}+\frac{b}{a} \right )\geq 6$

 

BT6: ( Vasile Cirtoaje

Cho a,b,c là các số thực dương , CMR :$\sum \sqrt{\frac{2a}{a+b}}\leq 3$

 

Lời giải : 

Tương tự : 

$\Leftrightarrow \sum \left (\sqrt{\frac{2a}{a+b}}+ \sqrt{\frac{2b}{a+b}} \right )\leq 6$

Đúng vì theo Cauchy - Schwarz : 

$\sqrt{\frac{2a}{a+b}}+\sqrt{\frac{2b}{a+b}}\leq \sqrt{2.\left ( \frac{2a}{a+b}+\frac{2b}{a+b} \right )}=2$

 

BT7: ( Vasile Cirtoaje - Arqady

 

Cho a,b,c là các số thực dương. CMR : 

$\sum \frac{a^2}{a+b}\leq \frac{3(a^2+b^2+c^2)}{2(a+b+c)}$

 

Lời giải : 

Tương tự, ta phải chứng minh : 

 

$\sum \left (\frac{a^2+b^2+c^2}{a+b+c}-\frac{a^2+b^2}{a+b}  \right )\geq 0\Leftrightarrow \sum \left ( \frac{c^2(a+b)-c(a+b)}{a+b} \right )\geq 0\Leftrightarrow \sum \left ( \frac{ac(a-c)}{a+b}-\frac{bc(b-c)}{a+b} \right )\geq 0\Leftrightarrow \sum \left ( \frac{ac(a-c)}{a+b}-\frac{ac(a-c)}{b+c} \right )\geq 0\Leftrightarrow \sum \frac{ac(a-c)^2}{(a+b)(b+c)}\geq 0$

 

Hiển nhiên đúng.

 

BT8: ( Vasc - Arqady

Cho a,b,c là các số thực dương. CMR : $\sum \frac{a}{\sqrt{a^2+b^2}}\leq \frac{3\sqrt{2}}{2}$

 

Bài này dễ nên các bạn tự giải được.

 

BT9: Cho a,b,c là các số thực dương. CMR : 

$\frac{a^2-c^2}{b+c}+\frac{b^2-a^2}{a+c}+\frac{c^2-b^2}{a+b}\geq 0$

 

Lời giải : 

 Tương tự như trên dẫn tới ta phải Chứng minh : 

 

$\sum \left ( \frac{a^2-c^2}{b+c}+\frac{c^2-a^2}{a+b} \right )\geq 0\Leftrightarrow \sum \frac{(a+c)(a-c)^2}{(b+c)(a+b)}\geq 0$

 

Hiển nhiên đúng.

 

BT10: ( Moldova TST 2006

Cho a,b,c là 3 cạnh của một tam giác. CMR : 

$\sum a^2\left ( \frac{b}{c}-1 \right )\geq 0$

 

Lời giải : 

BĐT có dạng : 

$\sum a^3b^2\geq abc(a+b+c)$

Ta phải chứng minh  : 

$\sum (a^3b^2+a^2b^3)\geq 2abc(a^2+b^2+c^2)\Leftrightarrow \sum a^3(b^2+c^2)\geq \sum 2a^3bc$

 

Hiển nhiên đúng theo BĐT AM - GM

 

BT11 : ( Micrea Lascu, Bandila, Panaitopol

Cho a,b,c là các số thực, CMR : 

$a(a+b)^3+b(b+c)^3+c(c+a)^3\geq 0$

 

Quá dễ nếu bạn sử dụng S.O.F

 

BT12 : ( Phan Thành Nam - VMEO 2005

Cho a,b,c là các số thực dương. CMR : 

 

$\sum \frac{1}{a\sqrt{a+b}}\geq \frac{3}{\sqrt{2abc}}$

 

Lời giải : 

Tương tự ta đưa bài toán về phải chứng minh : 

 

$\sum \left ( \frac{1}{a\sqrt{a+b}}+\frac{1}{b\sqrt{a+b}} \right )\geq \frac{3\sqrt{2}}{\sqrt{abc}}\Leftrightarrow \sum \frac{\sqrt{a+b}}{ab}\geq \frac{3\sqrt{2}}{\sqrt{abc}}\Leftrightarrow \sum c\sqrt{a+b}\geq 3\sqrt{2}\sqrt{abc}$

 

Đúng bằng cách sử dụng trực tiếp BĐT AM - GM 2 lần.

 

BT13: ( Gabriel Dospinescu )

Cho a,b,c là các số thực dương. CMR : 

 

$\sum \frac{1}{4a}+\sum \frac{1}{a+b}\geq 3\sum \frac{1}{3a+b}$

 

Lời giải : 

 

Tương tự như các bài toán trước ta đưa BĐT về đối xứng và ta phải chứng minh : 

$\sum \frac{1}{2a}+2\sum \frac{1}{a+b}\geq 3\sum  \left (\frac{1}{3a+b}+\frac{1}{3b+a}  \right )$

 

Hay 

$\sum \left ( \frac{1}{4a}+\frac{1}{4b}+\frac{2}{a+b}-\frac{3}{3a+b}-\frac{3}{3b+a} \right )\geq 0\Leftrightarrow \frac{1}{4}\sum \left ( (\frac{1}{a}+\frac{1}{b}-\frac{4}{a+b})-12(\frac{1}{3a+b}+\frac{1}{3b+a}-\frac{1}{a+b}) \right )\geq 0\Leftrightarrow \sum \frac{(a-b)^2}{a+b}.\frac{3a^2+3b^2-2ab}{ab(3a+b)(3b+a)}\geq 0$

 

 

Hiển nhiên đúng !

Hay ĐPCM

 

BT14 : (Iran MO 2005

 

Cho a,b,c là các số thực dương. CMR : 

 

$\left ( \frac{a}{b}+\frac{b}{c}+\frac{c}{a} \right )^2\geq (a+b+c)\left ( \frac{1}{a}+\frac{1}{b}+\frac{1}{c} \right )$

 

Lời giải : 

 

Ta sẽ phải chứng minh : 

 

$\sum \frac{a^2}{b^2}+\sum \frac{a}{c}\geq 3+\sum \frac{a}{b}$$\sum \left (\frac{a^2}{b^2} +\frac{b^2}{a^2} \right )+\sum \left ( \frac{a}{c}+\frac{c}{a}\right )\geq 3+3+\sum \left ( \frac{a}{b}+\frac{b}{a} \right )\Leftrightarrow \sum \left ( \frac{a^2}{b^2}+\frac{b^2}{a^2} \right )\geq 6$

Hiển nhiên đúng theo BĐT AM - GM 

 

 

BT15 : Cho a,b,c là các số thực dương. CMR : 

 

$\sum \frac{a}{\sqrt{a+b}}\geq \frac{\sqrt{a}+\sqrt{b}+\sqrt{c}}{\sqrt{2}}$

 

Lời giải : 

 

Ta phải chứng minh : 

$\sum \left ( \frac{a}{\sqrt{a+b}}+\frac{b}{\sqrt{a+b}} \right )\geq \sqrt{2}(\sqrt{a}+\sqrt{b}+\sqrt{c})\Leftrightarrow \sum \sqrt{a+b}\geq \sqrt{2}(\sqrt{a}+\sqrt{b}+\sqrt{c})$

 

Hiển nhiên đúng vì theo BĐT AM - GM với x,y là các số dương : $\sqrt{x}+\sqrt{y}\leq \sqrt{2(x+y)}$




#554223 Chứng minh rằng:$a^{3}+b^{3}+c^{3}=3abc$

Posted by duythanbg on 15-04-2015 - 20:04 in Đại số

Dễ . Ta chỉ cần xét a,b,c đôi một phân biệt.

 

Nếu x + y = 1 . Thay y = 1 - x vào phương trình 1 : 

 

ax - bx + b - c = 0

 

Rút ra x theo a,b,c  rồi thay vào phương trình 2 ta sẽ có : 

 

$a^2+b^2+c^2-ab-bc-ca=0$ 

 

....




#549600 $a^4+ b^4\leq \frac{a^6}{b^2}+\frac{b^6}{a^2}$

Posted by duythanbg on 26-03-2015 - 18:45 in Bất đẳng thức và cực trị

3.

 

Ta có : 

 

$a^3+a^3+b^3\geq 3a^2b$

 

Tương tự rồi cộng các BĐT lại ta có ĐPCM.




#549598 $a^2(1+b^2)+b^2(1+c^2)+c^2(1+a^2)\geqslant 6abc$

Posted by duythanbg on 26-03-2015 - 18:43 in Bất đẳng thức và cực trị

bạn có thể làm rõ được không?

$\frac{a^2}{4}+b^2\geq ab$

$\frac{a^2}{4}+c^2\geq ac$

$\frac{a^2}{4}+d^2\geq ad$

$\frac{a^2}{4}+e^2\geq ae$

 

Cộng lại ta có ĐPCM.




#549596 $a^2(1+b^2)+b^2(1+c^2)+c^2(1+a^2)\geqslant 6abc$

Posted by duythanbg on 26-03-2015 - 18:30 in Bất đẳng thức và cực trị

2.

 

Ta có : 

$\frac{a^2}{4}+b^2\geq ab$ 

 

Tương tự rồi cộng lại ta có ĐPCM .




#543546 $\frac{1}{x^{2}}+\frac{1...

Posted by duythanbg on 09-02-2015 - 18:57 in Phương trình - Hệ phương trình - Bất phương trình

LHS :  left hand side = vế trái 




#543024 $\sum \sqrt{\frac{a^{2}}{4a...

Posted by duythanbg on 04-02-2015 - 22:15 in Bất đẳng thức - Cực trị

anh ơi em không biết BĐT Vasc , anh có thể ghi ra được không

 

đây là 1 chuyên đề nhỏ về BĐT Vasc ( Internet ) 

 

khá là hay . 

Attached Files




#543003 Cho a,b,c >0 thỏa mãn a+b+c = 6abc.

Posted by duythanbg on 04-02-2015 - 21:15 in Bất đẳng thức và cực trị

1. 

 

Áp dụng trực tiếp BĐT Cauchy - Bunyakovsky - Schwarz ta có : 

 

 

$\left ( 2a^2+b^2 \right )\left ( 2a^2+c^2 \right )=(a^2+a^2+b^2)(a^2+c^2+a^2)\geq (a^2+ac+ab)^2=a^2(a+b+c)^2$

 

Tương tự và do đó : 

 

$VT\leq \sum \frac{a}{(a+b+c)^2}=\frac{1}{3}$

 

(Vì a + b + c = 3 ) 




#542907 CHứng minh các bất đẳng thức

Posted by duythanbg on 03-02-2015 - 22:22 in Bất đẳng thức - Cực trị

  do bất đẳng thức dài nên em không làm tiêu đề được,mong mod thông cảm

1, cho a,b,c là các số không âm.chứng minh $a(-\sqrt{a}+\sqrt{b}+\sqrt{c})+b(-\sqrt{b}+\sqrt{c}+\sqrt{a})+c(-\sqrt{c}+\sqrt{b}+\sqrt{a})\leq 3\sqrt{abc}$

2, cho a,b,c đôi một khác nhau.Chứng minh $\frac{a^{3}-b^{3}}{(a-b)^{3}}+\frac{b^{3}-c^{3}}{(b-c)^{3}}+\frac{c^{3}-a^{3}}{(c-a)^{3}}\geq \frac{9}{4}$

3, cho a,b,c,d>0. Chứng minh $\frac{1}{a^{4}+b^{4}+c^{4}+abcd}+\frac{1}{b^{4}+c^{4}+d^{4}+abcd}+\frac{1}{c^{4}+d^{4}+a^{4}+abcd}\frac{1}{d^{4}+a^{4}+b^{4}+abcd}\leq \frac{1}{abcd}$

 

1, Đặt ẩn phụ : 

 

$\sqrt{a}=x;\sqrt{b}=y;\sqrt{c}=z$

 

BĐT đã cho sẽ có dạng tương đương là : 

 

$a^3+b^3+c^3+3abc\geq \sum ab(a+b)$

 

Là BĐT Schur bậc 1 ,,, Đúng .

 

:icon6:




#542785 Bất đẳng thức phụ

Posted by duythanbg on 02-02-2015 - 22:35 in Bất đẳng thức và cực trị

cái này còn suy ra đc 1 đống bất đẳng thức tg tự hay sao ý ???

 

VD với 3 số, (tương tự tổng quát cho n số ) : Cho $a,b,c\geq 1$ CMR : 

 

$\frac{1}{a^3+1}+\frac{1}{b^3+1}+\frac{1}{c^3+1}\geq \frac{3}{1+abc}$

 

LG :

 

Áp dụng liên tiếp 2 lần BĐT với 2 số đã chứng minh ở trên :

 

$\frac{1}{a^3+1}+\frac{1}{b^3+1}+\frac{1}{c^3+1}+\frac{1}{abc+1}\geq \frac{2}{1+\sqrt{a^3b^3}}+\frac{2}{1+\sqrt{abc^4}}\geq \frac{4}{1+\sqrt{\sqrt{a^4b^4c^4}}}=\frac{4}{1+abc}$

 

Suy ra ĐPCM.

 

TQ: Cho $a_{i}\geq 1\forall i=\overline{1,n}$

CMR : 

$\sum_{n}^{i=1}\frac{1}{1+a^n}\geq \frac{n}{1+\coprod_{n}^{i=1}a_{i}}$




#542763 $\sum \frac{1}{1+a^{4}}=1$...

Posted by duythanbg on 02-02-2015 - 21:18 in Bất đẳng thức và cực trị

Ta có : 

 

$\frac{1}{1+a^4}+\frac{1}{1+c^4}+\frac{1}{1+d^4}=1-\frac{1}{1+c^4}=\frac{c^4}{1+c^4}\geq \frac{3}{\sqrt[3]{(1+a^4)(1+b^4)(1+c^4)}}$

 

Viết các BĐT tương tự rồi nhân lại ta sẽ có 

 

$a^4b^4c^4d^4\geq 81\Rightarrow ....$